Question

In: Physics

1. Unpolarized light of intensity I0 is incident on three polarizing filters. The axis of the...

1. Unpolarized light of intensity I0 is incident on three polarizing filters. The axis of the first is vertical, that of the second is 29° from vertical, and that of the third is horizontal. What light intensity emerges from the third filter?  ✕ I0

2. A 215 mW horizontally polarized laser beam passes through a polarizing filter whose axis is 23° from vertical. What is the power of the laser beam as it emerges from the filter? in mW

3. Two narrow slits 44 µm apart are illuminated with light of wavelength 600 nm. What is the angle of the m = 2 bright fringe in radians? what is the angle in degrees?

4. A diffraction grating is illuminated simultaneously with red light of wavelength 670 nm and light of an unknown wavelength. The fifth-order maximum of the unknown wavelength exactly overlaps the fourth-order maximum of the red light. What is the unknown wavelength? in nm

Solutions

Expert Solution

1)      After the initially unpolarized light passes through the first sheet, the intensity of the light when it emerges from that sheet is

I0 / 2

. Then after it passes through the second, sheet the intensity becomes

( I0/2)x(cos(29))2. For the last sheet, the angle between the polarized light and the direction of polarization is (90 - 29)=61degrees.

So, the intensity when it emerges from the third sheet is .2350(I0 / 2)x(cos(61))2

. After approximating (cos(61))2, you should get that the intensity is approximately .17975*(I0 / 2).

2)     P=215 cos(23)

         =197.9 mw

3) Displacement y = (Order m x Wavelength x Distance D)/(slit separation d)

m=2
For double slit separation d =44 micrometers

and light wavelength λ =600 nm

on a screen at distance D = 100 cm

the displacement from the centerline for maximum intensity will be

y = m λ D / d

=(2x600x10-9x100)/(44x10-6)

=2.727272 cm
This corresponds to an angle of θ

4)   d sinθ =m λ

So, dsinθred = 4x670nm

Also, d sin θ = 5λ

However, if they overlap, then

dsinθred=d sinθ?

4x670nm=5λ?

Therefore λ=(4x670nm)/5

=536 nm


Related Solutions

1. Unpolarized light of intensity I0 is incident on three polarizing filters. The axis of the...
1. Unpolarized light of intensity I0 is incident on three polarizing filters. The axis of the first is vertical, that of the second is 31° from vertical, and that of the third is horizontal. What light intensity emerges from the third filter? ✕ I0 2. A 215 mW horizontally polarized laser beam passes through a polarizing filter whose axis is 28° from vertical. What is the power of the laser beam as it emerges from the filter? mW 3. Two...
An unpolarized beam of light is incident on a pair of polarizing filters. The first filter...
An unpolarized beam of light is incident on a pair of polarizing filters. The first filter set to polarize in the vertical direction and the second filter is set to polarize 38° from vertical. What is the intensity of the light beam after the second filter relative to the original beam?
Unpolarized light is passed through three successive polarizing filters, each with its transmission axis at 45...
Unpolarized light is passed through three successive polarizing filters, each with its transmission axis at 45 to the preceding filter. What percentage of the total light striking the first filter gets through the last filter? Does the transmission depend on wavelength? Explain.
Unpolarized light is passed through three successive polarizing filters, each with its transmission axis at 45deg...
Unpolarized light is passed through three successive polarizing filters, each with its transmission axis at 45deg to the preceding filter. What percentage of the total light striking the first filter gets through the last filter? Does the transmission depend on wavelength? Explain.
Light of original intensity I0 passes through two ideal polarizing filters having their polarizing axes oriented...
Light of original intensity I0 passes through two ideal polarizing filters having their polarizing axes oriented as shown in the figure (Figure 1). You want to adjust the angle ϕ so that the intensity at point P is equal to I0/8.00. A.)If the original light is unpolarized, what should ϕbe? B.)If the original light is linearly polarized in the same direction as the polarizing axis of the first polarizer the light reaches, what should ϕ be?
Light of intensity I0 and polarized parallel to the transmission axis of a polarizer is incident...
Light of intensity I0 and polarized parallel to the transmission axis of a polarizer is incident on an analyzer. (a) If the transmission axis of the analyzer makes an angle of 46° with the axis of the polarizer, what is the intensity of the transmitted light? I0 (b) What should the angle between the transmission axes be to make I/I0 = 5/8? °
Linearly polarized light that is oriented along the x-axis and has intensity I0 is incident on...
Linearly polarized light that is oriented along the x-axis and has intensity I0 is incident on a linear polarizer (LP1) with transmission axis oriented at an angle θ1=15° clockwise with respect to the x axis. The light then passes through a second linear polarizer (LP2) with transmission axis oriented at an angle θ2 = 45° counterclockwise relative to the x axis as shown before passing through a quarter wave plate. The fast axis of the quarter wave plate is aligned...
Linearly polarized light that is oriented along the x-axis and has intensity I0 is incident on...
Linearly polarized light that is oriented along the x-axis and has intensity I0 is incident on a linear polarizer (LP1) with transmission axis oriented at an angle θ1=15° clockwise with respect to the x axis. The light then passes through a second linear polarizer (LP2) with transmission axis oriented at an angle θ2 = 45° counterclockwise relative to the x axis as shown before passing through a quarter wave plate. The fast axis of the quarter wave plate is aligned...
Linearly polarized light that is oriented along the x-axis and has intensity I0 is incident on...
Linearly polarized light that is oriented along the x-axis and has intensity I0 is incident on a linear polarizer (LP1) with transmission axis oriented at an angle θ1=15° clockwise with respect to the x axis. The light then passes through a second linear polarizer (LP2) with transmission axis oriented at an angle θ2 = 45° counterclockwise relative to the x axis as shown before passing through a quarter wave plate. The fast axis of the quarter wave plate is aligned...
Unpolarized light of intensity Io is incident on a polarizer whose transmission axis makes an angle...
Unpolarized light of intensity Io is incident on a polarizer whose transmission axis makes an angle of zero degrees with the vertical. A second polarizing plate is placed after the first. The transmission axis of the second plate makes an angle phi with the vertical. a) sketch a graph of the transmitted intensity vs the angle phi. your angle should vary from 0 to pi/2. b) at what value of phi does the slope of your graph have its greatest...
ADVERTISEMENT
ADVERTISEMENT
ADVERTISEMENT